RegistrierenRegistrieren   LoginLogin   FAQFAQ    SuchenSuchen   
Lorentztransformation in CM frame, Teilchenphysik
 
Neue Frage »
Antworten »
    Foren-Übersicht -> Quantenphysik
Autor Nachricht
SuperVektor



Anmeldungsdatum: 07.02.2023
Beiträge: 1

Beitrag SuperVektor Verfasst am: 07. Feb 2023 17:28    Titel: Lorentztransformation in CM frame, Teilchenphysik Antworten mit Zitat

Meine Frage:
Angenommen, wir observieren in einem Teilchenbeschleuniger die Reaktion:

Bekannt sind die Energie-Impuls-Vierervektoren der Teilchen a, b und c in natürlichen Einheiten. Ich möchte das Teilchen c mittels Lorentz-Transformation in das Schwerpunktsystem überführen und mir die -Verteilung angucken. Wie genau würde ich die Transformation durchführen?

Alles, was ich bisher im Internet gefunden habe, bezieht sich auf Spezialfälle, in denen nur entlang einer Koordinatenachse transformiert wird. Ich möchte mir den allgemeinen Fall angucken, in dem entlang jeder Achse transformiert wird.

Weiterhin wird häufig mit der relativen Geschwindigkeit der Bezugssysteme argumentiert - ich weiß jedoch nicht, wie ich dieses Konzept auf Energie-Impuls-Vierervektoren anweden würde, denn es ist ja keine Geschwindigkeit gegeben, nur Impulse. Wie würde ich in diesem Fall und berechnen?

Vielen Dank an alle!

Meine Ideen:
Ich gehe davon aus, man muss die invariante Masse des Anfangszustands für LAB- und CM-System gleichsetzen. Im LAB-System ist diese bekannt, so kann man die entgegengesetzen Impulse im CM-System berechnen. Ich verstehe nur nicht, wie genau sich die individuellen x-, y- und z-Orientierungen des Impulses transformieren. Weiterhin weiß ich nicht genau, wie die Lorentzmatrix für eine Transformation entlang aller Achsen aussehen müsste.

Es geht hier übrigens nicht um eine Hausaufgabe, keine Angst Augenzwinkern
SuperVektor2
Gast





Beitrag SuperVektor2 Verfasst am: 10. Feb 2023 00:18    Titel: Antworten mit Zitat

Meine Frage wurde beantwortet, die Lösung kann man auf "Physics stack exchange" finden, sucht nach "General Lorentz boost of four-momentum in CM frame, particle physics".

(Ich kann leider keine URLs posten)
as_string
Moderator


Anmeldungsdatum: 09.12.2005
Beiträge: 5786
Wohnort: Heidelberg

Beitrag as_string Verfasst am: 10. Feb 2023 10:31    Titel: Antworten mit Zitat

Danke für die Info!
https://physics.stackexchange.com/questions/749036

Gruß
Marco
Neue Frage »
Antworten »
    Foren-Übersicht -> Quantenphysik